0% found this document useful (0 votes)
12 views10 pages

Final Exam Solutions-1

The document contains solutions to various physics problems related to NEET JEE, including calculations for the sum of whole numbers, time periods of pendulums at different heights, and derivatives of logarithmic functions. It also discusses the motion of particles, the de Broglie wavelength, potential energy, and gravitational weight at different depths. Each problem is followed by a detailed solution and the correct answer option.

Uploaded by

vasan14775
Copyright
© © All Rights Reserved
We take content rights seriously. If you suspect this is your content, claim it here.
Available Formats
Download as PDF, TXT or read online on Scribd
0% found this document useful (0 votes)
12 views10 pages

Final Exam Solutions-1

The document contains solutions to various physics problems related to NEET JEE, including calculations for the sum of whole numbers, time periods of pendulums at different heights, and derivatives of logarithmic functions. It also discusses the motion of particles, the de Broglie wavelength, potential energy, and gravitational weight at different depths. Each problem is followed by a detailed solution and the correct answer option.

Uploaded by

vasan14775
Copyright
© © All Rights Reserved
We take content rights seriously. If you suspect this is your content, claim it here.
Available Formats
Download as PDF, TXT or read online on Scribd
You are on page 1/ 10

NEET JEE Physics

Foundations and Fundamentals

Day 10: Final Exam


Solutions

1. Find the sum of the first n whole numbers.


n(n+1) (n−1)n (n+1)(n+2) (n+1)(n−1)
(a) 2
(b) 2
(c) 2
(d) 2

Solution: The first n whole numbers are:

0 + 1 + 2 + ··· + n − 1

So this this of n-1 natural numbers


(n − 1)(n)
S=
2

(n−1)n
Correct option: (b) 2

2. A simple pendulum performs small oscillations at a height R above the Earth’s surface and
has a time period T1 = 4 s. What will be its time period T2 if it is taken to a point at height 2R
from the Earth’s surface?
Choose the correct relation between T1 and T2 , where R is the radius of the Earth.
(a) 2T1 = T2 (b) 2T1 = 3T2 (c) T1 = T2 (d) 3T1 = 2T2

Hint: Time period of a pendulum is inversely proportional to the square root of gravitational
acceleration and the gravitational acceleration inversely is proportional to square of distance
from the centre.
Solution: The time period of a pendulum is given by:
1
T ∝√
g

And the acceleration due to gravity at a height h from the surface of the Earth is:
1
gh ∝
(R + h)2

So the time period becomes:


1 p
T ∝ √ ∝ (R + h)2 = R + h
g

Let’s compare the time periods at two different heights:

Page 1 of 10
Step 1: At height R:
T1 ∝ R + R = 2R

Step 2: At height 2R:


T2 ∝ R + 2R = 3R

Now, take the ratio:


T1 2R 2
= = ⇒ 3T1 = 2T2
T2 3R 3

Correct Option: (d) 3T1 = 2T2

3. Evaluate the definite integral: Z 2π


1 x
sin2 dx
0 2 2
π
(a) π (b) 2π (c) 2
(d) 0
Solution: We are given: Z 2π
1 x
sin2 dx
0 2 2

Use the identity:


x 1 − cos x 1  x  1 − cos x
sin2 = ⇒ sin2 =
2 2 2 2 4

So the integral becomes:


Z 2π 2π
1 − cos x
Z
1
dx = (1 − cos x) dx
0 4 4 0

Now compute: Z 2π Z 2π
1 dx = 2π, cos x dx = 0
0 0

Therefore:

1 − cos x
Z
1 π
dx = · (2π − 0) =
0 4 4 2
π
Correct option: (c) 2

4. Find the derivative of log(xx ) with respect to x:


1 log x 1+log x
(a) x
(b) log x + 1 (c) x
(d) x

Solution: We are given:


y = log(xx )

Use logarithmic identity:


log(xx ) = x log x

Differentiate using the product rule:


d 1
[x log x] = 1 · log x + x · = log x + 1
dx x
Page 2 of 10
So the derivative is:
d
log(xx ) = log x + 1
dx
Correct option: (b) log x + 1

5. The coordinates of a particle moving in the x–y plane are given by:

x = 2 + 4t, y = 3t + 8t2

The motion of the particle is:


(a) uniformly accelerated having motion along a parabolic path
(b) uniform motion along a straight line
(c) uniformly accelerated having motion along a straight line
(d) non-uniformly accelerated motion
Solution: We are given:
x(t) = 2 + 4t and y(t) = 3t + 8t2

Let us analyze motion in the x- and y-directions separately:


1. Motion in x-direction:
dx d2 x
x = 2 + 4t ⇒ =4 (constant velocity) ⇒ =0
dt dt2
So, no acceleration in the x-direction.
2. Motion in y-direction:

dy d2 y
y = 3t + 8t2 ⇒ = 3 + 16t ⇒ 2 = 16 (constant acceleration)
dt dt

Hence, the particle has uniform acceleration in the y-direction and constant velocity
in the x-direction.
Also, to find the path, eliminate t:
   2
x−2 x−2 x−2
t= ⇒ y=3 +8
4 4 4

This is a quadratic in x, so the path is a parabola.


Correct option: The motion is uniformly accelerated along a parabolic path.
Correct option: (a) uniformly accelerated having motion along a parabolic path

Page 3 of 10
6. The graph which shows the variation of the de Broglie wavelength λ of a particle with its
associated momentum p is:

λ λ

p p
(a) (b)

λ λ

p p
(c) (d)

Hint: The de Broglie wavelength is inversely proportional to the momentum. Answer: Op-

tion (4)

Solution:
The de Broglie wavelength of a particle is given by the relation:

h
λ=
p

where h is Planck’s constant and p is the momentum of the particle.


This implies:

1
λ∝
p

This indicates that the λ decreased as p increases.


Hence, the correct graph is shown in Option (4).

7. The potential energy of a system is given by:

U (x) = (x + 2)2 − 4 J

Which of the following statements are correct:


1. The minimum value of potential energy is −4 J.
2. At x = −2 m, potential energy is minimum.
3. At x = 0, potential energy is minimum.

Page 4 of 10
4. At x = 2, potential energy is zero.
(a) 1,2 (b) 1,3 (c) only 3 (d) None
Solution: We are given the potential energy function:
U (x) = (x + 2)2 − 4

Step 1: Compute the derivative

dU
= 2(x + 2)
dx
Step 2: Find the value of x when the derivative is zero
Set the derivative to zero:
2(x + 2) = 0 ⇒ x = −2

Step 3: Use second derivative test

d2 U
=2>0
dx2
Since the second derivative is positive, the function has a local minimum at x = −2.
Step 4: Evaluate potential energy

U (−2) = (−2 + 2)2 − 4 = 0 − 4 = −4 J

Step 5: Check other options


(1) True. Minimum value is −4 J.
(2) True. Minimum occurs at x = −2.
(3) False. At x = 0, U = (0 + 2)2 − 4 = 4 − 4 = 0 J, not minimum.
(4) False. At x = 2, U = (2 + 2)2 − 4 = 16 − 4 = 12 J, not zero.
Correct option: The correct statements are 1 and 2.
Correct option: (a)

8. Assuming the Earth to be a sphere of uniform mass density, a body weighs 300 N on the
surface of the Earth. How much would it weigh at a depth R4 below the surface of the Earth?
Hint: The weight of an object is proportional to the mass enclosed beneath it and inversely
proportional to the square of its distance from the center of the Earth.
(a) 75 N (b) 225 N (c) 300 N (d) 375 N
Solution: Let the radius of the Earth be R, and the depth from the surface be d. Then the
distance from the center is:
r =R−d

Step 1: Mass enclosed at depth r


For a sphere of uniform density ρ, the mass enclosed within radius r is:
4
Mr = ρ · πr3
3
Page 5 of 10
Step 2: Gravitational field at depth r
The gravitational field at this depth is:

Mr ρ · 43 πr3 4πρ
g0 = G · 2
= G · 2
=G· ·r
r r 3

So,
g0 ∝ r = R − d

Step 3: Ratio of gravity at depth to surface:

g0
 
R−d 0 d
= ⇒g =g 1−
g R R

Step 4: Weight at depth


Since weight is proportional to gravitational acceleration:
 
0 d
W =W 1−
R

Given:
R
W = 300 N, d=
4
Substitute:  
0 1 3
W = 300 1 − = 300 · = 225 N
4 4
R
Correct option: The body would weigh 225 N at a depth 4
below the surface.
Correct option: (b)

9. Find the value of tan2 (375◦ ).


√ √
2− 3 √ 2+ 3 √
(a) √ (b) 7 − 4 3 (c) √ (d) 7 + 4 3
2+ 3 2− 3
Solution: Use the periodicity identity:

tan(θ) = tan(θ − 360◦ )

Then apply the tangent subtraction identity:


tan A − tan B
tan(A − B) =
1 + tan A tan B

We are asked to compute:


tan2 (375◦ )

Step 1: Use periodicity

375◦ = 360◦ + 15◦ ⇒ tan(375◦ ) = tan(15◦ ) ⇒ tan2 (375◦ ) = tan2 (15◦ )

Page 6 of 10
Step 2: Use the identity
tan 45◦ − tan 30◦
tan(15◦ ) = tan(45◦ − 30◦ ) =
1 + tan 45◦ · tan 30◦

We know:
1
tan 45◦ = 1, tan 30◦ = √
3

√1 3−1 √

1− 3

3 3−1
tan(15 ) = = =√
1+1 · √13 1 + √13 3+1

Step 3: Square the result

√ !2 √
3−1 ( 3 − 1)2
tan2 (15◦ ) = √ = √
3+1 ( 3 + 1)2

√ √ √
( 3 − 1)2 = 3 − 2 3 + 1 = 4 − 2 3
√ √ √
( 3 + 1)2 = 3 + 2 3 + 1 = 4 + 2 3


2 4−2 3

tan (15 ) = √
4+2 3

Now multiply numerator and denominator by the conjugate of the denominator:

√ √ √ √ √
(4 − 2 3)(4 − 2 3) (4 − 2 3)2 16 − 16 3 + 12 28 − 16 3 √
= √ √ = √ = = =7−4 3
(4 + 2 3)(4 − 2 3) 16 − (2 3)2 16 − 12 4

Correct option: √
(b) 7 − 4 3

10. Consider the quadratic equation:

x2 − 200000002x + 10000000200000001 = 0

What are the roots of the equation?


(a) x = 100000001, 100000001 (b) x = 100000000, 100000002
(c) x = 400000004, −200000002 (d) The roots are irrational
Solution: Let the roots be x1 and x2 .
We are given the quadratic:

x2 − 200000002x + 10000000100000001 = 0

Then:
x1 + x2 = 200000002 (sum of roots)
x1 x2 = 10000000200000001 (product of roots)

Page 7 of 10
Step 1: Use odd even argument argument
The product x1 x2 is odd, so both x1 and x2 must be odd.
Among the options, only option (a) has both roots odd:

x1 = x2 = 100000001 (odd)

Step 2: Verify the sum

x1 + x2 = 100000001 + 100000001 = 200000002

Step 3: Verify the product using identity

x1 x2 = 1000000012 = (108 + 1)2 = 1016 + 2 · 108 + 1


= 10000000000000000 + 200000000 + 1
= 10000000200000001

Correct option:
(a) x = 100000001, 100000001

11. If θ = 143◦ , what is the value of tan θ likely to be?


(a) A positive number less than 1 (b) A negative number greater than 1
(c) A negative number less than 1 (d) A positive number greater than 1
Solutions: Recall the signs of trigonometric functions in different quadrants:
- tan θ is positive in the 1st and 3rd quadrants
- tan θ is negative in the 2nd and 4th quadrants
Also, use the concept of the reference angle:

Reference angle = 180◦ − 143◦ = 37◦

Since θ = 143◦ lies in the 2nd quadrant, and tangent is negative there, we expect:

tan(143◦ ) = − tan(37◦ )

Since θ is less than 45◦ , tan(θ) is less than 1


This is a negative number less than 1 in magnitude.
Correct option:
(c) A negative number less than 1

12. Consider the binomial expansion of (2a + 3b)8 . What is the ratio of the coefficients of the 4th
T4 term to the 3rd T3 term?
Hint: Use the binomial coefficient formula:

Tr+1 = nCr xn−r y r

1 1
(a) 3 (b) 2
(c) 2 (d) 3

Page 8 of 10
Solution: In the binomial expansion of (2a + 3b)8 , the general term is:

Tr+1 = 8Cr (2a)8−r (3b)r

So, the:
• 3rd term corresponds to r = 2 ⇒ T3 = 8C2 (2a)6 (3b)2
• 4th term corresponds to r = 3 ⇒ T4 = 8C3 (2a)5 (3b)3
We are asked to find the ratio of just the binomial coefficients:

8
C3 25 33 56 · 3
Ratio = 8C 26 32
= =3
2 28 · 2

Correct option:
(a) 3

13. The line passes through the points A(2, 5) and B(6, 13). Which of the following points lies on
the same line?
(a) (4, 8) (b) (5, 11) (c) (8, 18) (d) (10, 20)
Solution: From points A(2, 5) and B(6, 13), compute the slope:
13 − 5 8
m= = =2
6−2 4

Using point-slope form at A:

y − 5 = 2(x − 2) ⇒ y = 2x + 1

Now check which point satisfies this:


- (a): y = 8, 2x + 1 = 2(4) + 1 = 9 6= 8
- (b): y = 11, 2(5) + 1 = 11
- (c): y = 18, 2(8) + 1 = 17 6= 18
- (d): y = 20, 2(10) + 1 = 21 6= 20
Correct option:
(b) (5, 11)

14. Consider the equation:


x2 + y 2 − 6x + 4y + 9 = 0
Which of the following geometric figures does this equation represent?
(a) A parabola (b) A circle (c) An ellipse (d) A Hyperbola
Solution: Given:
x2 + y 2 − 6x + 4y + 9 = 0

Group and complete the square:

(x2 − 6x) + (y 2 + 4y) + 9 = 0

Page 9 of 10
Complete the square in both groups:

(x − 3)2 − 9 + (y + 2)2 − 4 + 9 = 0
(x − 3)2 + (y + 2)2 − 4 = 0
(x − 3)2 + (y + 2)2 = 4

This is the equation of a circle with:


- Center: (3, −2)

- Radius: 4 = 2
Correct option:
(b) A circle

15. Charges are placed at equal angular positions on the circumference of circle of radius R,
and their magnitudes form a geometric progression:

1, 3, 9, 27, . . . , 39

What is the total electric potential at the center of the circle due to these charges? Hint:
Electric potential due to a point charge q at distance r is given by
kq
V =
r
The total potential is sum of potential of all charges
k k 310 −1 k 39 −1 k
(a) R
(310 − 1) (b) R
· 2
(c) R
· 2
(d) R
(39 − 1)
Solution: Each charge is at distance R from the center, so the potential at the center due
to each charge is:
k · qi
Vi =
R
Total potential:
9
k X i
Vtotal = 3
R i=0

This is a geometric series:


9
X 310 − 1 310 − 1
3i = =
i=0
3−1 2

So the total potential is:


k 310 − 1
V = ·
R 2
Correct option:
k 310 − 1
(b) ·
R 2

Page 10 of 10

You might also like